Why is $lim_{mto infty} frac{sinleft(frac{pi}{2^m}right)}{frac{pi}{2^m}} = 1$?












0














I am currently working on a proof with a good friend of mine that involves adding more and more triangles to the sides of a regular polygon but keeping the longest diagonal constant until eventually, it becomes a circle. And we ended up with this formula.



$4$-sided regular→$8$-sided regular→$16$-sided regular→$32$-sided regular$to ldots to n$-sided regular



(When $n$ tends to infinity, the area will be equal to that of a circle with the longest diagonal as diameter)



The Question is:



Can someone explain in detail:



Why does $$frac {sin frac {pi}{2^m}}{frac{pi}{2^m}} = 1$$ when $m$ tends to infinity?



Thanks in advance :D



Note I am still a beginner in finding limits. It would help greatly if you can explain step by step.










share|cite|improve this question









New contributor




dfd is a new contributor to this site. Take care in asking for clarification, commenting, and answering.
Check out our Code of Conduct.

























    0














    I am currently working on a proof with a good friend of mine that involves adding more and more triangles to the sides of a regular polygon but keeping the longest diagonal constant until eventually, it becomes a circle. And we ended up with this formula.



    $4$-sided regular→$8$-sided regular→$16$-sided regular→$32$-sided regular$to ldots to n$-sided regular



    (When $n$ tends to infinity, the area will be equal to that of a circle with the longest diagonal as diameter)



    The Question is:



    Can someone explain in detail:



    Why does $$frac {sin frac {pi}{2^m}}{frac{pi}{2^m}} = 1$$ when $m$ tends to infinity?



    Thanks in advance :D



    Note I am still a beginner in finding limits. It would help greatly if you can explain step by step.










    share|cite|improve this question









    New contributor




    dfd is a new contributor to this site. Take care in asking for clarification, commenting, and answering.
    Check out our Code of Conduct.























      0












      0








      0







      I am currently working on a proof with a good friend of mine that involves adding more and more triangles to the sides of a regular polygon but keeping the longest diagonal constant until eventually, it becomes a circle. And we ended up with this formula.



      $4$-sided regular→$8$-sided regular→$16$-sided regular→$32$-sided regular$to ldots to n$-sided regular



      (When $n$ tends to infinity, the area will be equal to that of a circle with the longest diagonal as diameter)



      The Question is:



      Can someone explain in detail:



      Why does $$frac {sin frac {pi}{2^m}}{frac{pi}{2^m}} = 1$$ when $m$ tends to infinity?



      Thanks in advance :D



      Note I am still a beginner in finding limits. It would help greatly if you can explain step by step.










      share|cite|improve this question









      New contributor




      dfd is a new contributor to this site. Take care in asking for clarification, commenting, and answering.
      Check out our Code of Conduct.











      I am currently working on a proof with a good friend of mine that involves adding more and more triangles to the sides of a regular polygon but keeping the longest diagonal constant until eventually, it becomes a circle. And we ended up with this formula.



      $4$-sided regular→$8$-sided regular→$16$-sided regular→$32$-sided regular$to ldots to n$-sided regular



      (When $n$ tends to infinity, the area will be equal to that of a circle with the longest diagonal as diameter)



      The Question is:



      Can someone explain in detail:



      Why does $$frac {sin frac {pi}{2^m}}{frac{pi}{2^m}} = 1$$ when $m$ tends to infinity?



      Thanks in advance :D



      Note I am still a beginner in finding limits. It would help greatly if you can explain step by step.







      limits trigonometry






      share|cite|improve this question









      New contributor




      dfd is a new contributor to this site. Take care in asking for clarification, commenting, and answering.
      Check out our Code of Conduct.











      share|cite|improve this question









      New contributor




      dfd is a new contributor to this site. Take care in asking for clarification, commenting, and answering.
      Check out our Code of Conduct.









      share|cite|improve this question




      share|cite|improve this question








      edited yesterday









      David K

      52.7k340115




      52.7k340115






      New contributor




      dfd is a new contributor to this site. Take care in asking for clarification, commenting, and answering.
      Check out our Code of Conduct.









      asked yesterday









      dfd

      1




      1




      New contributor




      dfd is a new contributor to this site. Take care in asking for clarification, commenting, and answering.
      Check out our Code of Conduct.





      New contributor





      dfd is a new contributor to this site. Take care in asking for clarification, commenting, and answering.
      Check out our Code of Conduct.






      dfd is a new contributor to this site. Take care in asking for clarification, commenting, and answering.
      Check out our Code of Conduct.






















          2 Answers
          2






          active

          oldest

          votes


















          2














          Because we have



          $$lim_{x to 0}frac{sin x}{x} = 1$$



          which is a standard limit typically proven through simple geometrical observations followed by the Squeeze Theorem. You can check this link for a various explanation of why this works, particularly the one by robjohn, which I find the simplest and most elegant.



          Another way of stating the above limit is



          $$sin x sim x; quad x to 0$$



          (Obviously, if $frac{sin x}{x} to 1$ as $x to 0$, then $sin x$ becomes very close to $x$.)



          In case of your limit, you have



          $$lim_{m to infty} frac{sinleft(frac{pi}{2^m}right)}{frac{pi}{2^m}}$$



          It’s clear that as $m to infty$, $frac{pi}{2^m} to 0$ due to the growing denominator, so we make a substitution $t = frac{pi}{2^m}$. Clearly, $t to 0$ as $m to infty$, so the limit becomes



          $$lim_{t to 0}frac{sin t}{t}$$



          which becomes $1$.






          share|cite|improve this answer





























            0














            If you replace $fracpi{2^m}$ with x, it is obvious that $x$ goes to zero when $m$ goes to infinity. So your limit is equivalent to:



            $$lim_{x to 0}frac{sin x}{x}$$



            ...which is 1. If you learn to evaluate just one limit, this is the one :) You can easily find several elementary proofs on the web and I really recommend this one from MSE.






            share|cite|improve this answer





















              Your Answer





              StackExchange.ifUsing("editor", function () {
              return StackExchange.using("mathjaxEditing", function () {
              StackExchange.MarkdownEditor.creationCallbacks.add(function (editor, postfix) {
              StackExchange.mathjaxEditing.prepareWmdForMathJax(editor, postfix, [["$", "$"], ["\\(","\\)"]]);
              });
              });
              }, "mathjax-editing");

              StackExchange.ready(function() {
              var channelOptions = {
              tags: "".split(" "),
              id: "69"
              };
              initTagRenderer("".split(" "), "".split(" "), channelOptions);

              StackExchange.using("externalEditor", function() {
              // Have to fire editor after snippets, if snippets enabled
              if (StackExchange.settings.snippets.snippetsEnabled) {
              StackExchange.using("snippets", function() {
              createEditor();
              });
              }
              else {
              createEditor();
              }
              });

              function createEditor() {
              StackExchange.prepareEditor({
              heartbeatType: 'answer',
              autoActivateHeartbeat: false,
              convertImagesToLinks: true,
              noModals: true,
              showLowRepImageUploadWarning: true,
              reputationToPostImages: 10,
              bindNavPrevention: true,
              postfix: "",
              imageUploader: {
              brandingHtml: "Powered by u003ca class="icon-imgur-white" href="https://imgur.com/"u003eu003c/au003e",
              contentPolicyHtml: "User contributions licensed under u003ca href="https://creativecommons.org/licenses/by-sa/3.0/"u003ecc by-sa 3.0 with attribution requiredu003c/au003e u003ca href="https://stackoverflow.com/legal/content-policy"u003e(content policy)u003c/au003e",
              allowUrls: true
              },
              noCode: true, onDemand: true,
              discardSelector: ".discard-answer"
              ,immediatelyShowMarkdownHelp:true
              });


              }
              });






              dfd is a new contributor. Be nice, and check out our Code of Conduct.










              draft saved

              draft discarded


















              StackExchange.ready(
              function () {
              StackExchange.openid.initPostLogin('.new-post-login', 'https%3a%2f%2fmath.stackexchange.com%2fquestions%2f3060629%2fwhy-is-lim-m-to-infty-frac-sin-left-frac-pi2m-right-frac-pi2%23new-answer', 'question_page');
              }
              );

              Post as a guest















              Required, but never shown

























              2 Answers
              2






              active

              oldest

              votes








              2 Answers
              2






              active

              oldest

              votes









              active

              oldest

              votes






              active

              oldest

              votes









              2














              Because we have



              $$lim_{x to 0}frac{sin x}{x} = 1$$



              which is a standard limit typically proven through simple geometrical observations followed by the Squeeze Theorem. You can check this link for a various explanation of why this works, particularly the one by robjohn, which I find the simplest and most elegant.



              Another way of stating the above limit is



              $$sin x sim x; quad x to 0$$



              (Obviously, if $frac{sin x}{x} to 1$ as $x to 0$, then $sin x$ becomes very close to $x$.)



              In case of your limit, you have



              $$lim_{m to infty} frac{sinleft(frac{pi}{2^m}right)}{frac{pi}{2^m}}$$



              It’s clear that as $m to infty$, $frac{pi}{2^m} to 0$ due to the growing denominator, so we make a substitution $t = frac{pi}{2^m}$. Clearly, $t to 0$ as $m to infty$, so the limit becomes



              $$lim_{t to 0}frac{sin t}{t}$$



              which becomes $1$.






              share|cite|improve this answer


























                2














                Because we have



                $$lim_{x to 0}frac{sin x}{x} = 1$$



                which is a standard limit typically proven through simple geometrical observations followed by the Squeeze Theorem. You can check this link for a various explanation of why this works, particularly the one by robjohn, which I find the simplest and most elegant.



                Another way of stating the above limit is



                $$sin x sim x; quad x to 0$$



                (Obviously, if $frac{sin x}{x} to 1$ as $x to 0$, then $sin x$ becomes very close to $x$.)



                In case of your limit, you have



                $$lim_{m to infty} frac{sinleft(frac{pi}{2^m}right)}{frac{pi}{2^m}}$$



                It’s clear that as $m to infty$, $frac{pi}{2^m} to 0$ due to the growing denominator, so we make a substitution $t = frac{pi}{2^m}$. Clearly, $t to 0$ as $m to infty$, so the limit becomes



                $$lim_{t to 0}frac{sin t}{t}$$



                which becomes $1$.






                share|cite|improve this answer
























                  2












                  2








                  2






                  Because we have



                  $$lim_{x to 0}frac{sin x}{x} = 1$$



                  which is a standard limit typically proven through simple geometrical observations followed by the Squeeze Theorem. You can check this link for a various explanation of why this works, particularly the one by robjohn, which I find the simplest and most elegant.



                  Another way of stating the above limit is



                  $$sin x sim x; quad x to 0$$



                  (Obviously, if $frac{sin x}{x} to 1$ as $x to 0$, then $sin x$ becomes very close to $x$.)



                  In case of your limit, you have



                  $$lim_{m to infty} frac{sinleft(frac{pi}{2^m}right)}{frac{pi}{2^m}}$$



                  It’s clear that as $m to infty$, $frac{pi}{2^m} to 0$ due to the growing denominator, so we make a substitution $t = frac{pi}{2^m}$. Clearly, $t to 0$ as $m to infty$, so the limit becomes



                  $$lim_{t to 0}frac{sin t}{t}$$



                  which becomes $1$.






                  share|cite|improve this answer












                  Because we have



                  $$lim_{x to 0}frac{sin x}{x} = 1$$



                  which is a standard limit typically proven through simple geometrical observations followed by the Squeeze Theorem. You can check this link for a various explanation of why this works, particularly the one by robjohn, which I find the simplest and most elegant.



                  Another way of stating the above limit is



                  $$sin x sim x; quad x to 0$$



                  (Obviously, if $frac{sin x}{x} to 1$ as $x to 0$, then $sin x$ becomes very close to $x$.)



                  In case of your limit, you have



                  $$lim_{m to infty} frac{sinleft(frac{pi}{2^m}right)}{frac{pi}{2^m}}$$



                  It’s clear that as $m to infty$, $frac{pi}{2^m} to 0$ due to the growing denominator, so we make a substitution $t = frac{pi}{2^m}$. Clearly, $t to 0$ as $m to infty$, so the limit becomes



                  $$lim_{t to 0}frac{sin t}{t}$$



                  which becomes $1$.







                  share|cite|improve this answer












                  share|cite|improve this answer



                  share|cite|improve this answer










                  answered yesterday









                  KM101

                  5,3691423




                  5,3691423























                      0














                      If you replace $fracpi{2^m}$ with x, it is obvious that $x$ goes to zero when $m$ goes to infinity. So your limit is equivalent to:



                      $$lim_{x to 0}frac{sin x}{x}$$



                      ...which is 1. If you learn to evaluate just one limit, this is the one :) You can easily find several elementary proofs on the web and I really recommend this one from MSE.






                      share|cite|improve this answer


























                        0














                        If you replace $fracpi{2^m}$ with x, it is obvious that $x$ goes to zero when $m$ goes to infinity. So your limit is equivalent to:



                        $$lim_{x to 0}frac{sin x}{x}$$



                        ...which is 1. If you learn to evaluate just one limit, this is the one :) You can easily find several elementary proofs on the web and I really recommend this one from MSE.






                        share|cite|improve this answer
























                          0












                          0








                          0






                          If you replace $fracpi{2^m}$ with x, it is obvious that $x$ goes to zero when $m$ goes to infinity. So your limit is equivalent to:



                          $$lim_{x to 0}frac{sin x}{x}$$



                          ...which is 1. If you learn to evaluate just one limit, this is the one :) You can easily find several elementary proofs on the web and I really recommend this one from MSE.






                          share|cite|improve this answer












                          If you replace $fracpi{2^m}$ with x, it is obvious that $x$ goes to zero when $m$ goes to infinity. So your limit is equivalent to:



                          $$lim_{x to 0}frac{sin x}{x}$$



                          ...which is 1. If you learn to evaluate just one limit, this is the one :) You can easily find several elementary proofs on the web and I really recommend this one from MSE.







                          share|cite|improve this answer












                          share|cite|improve this answer



                          share|cite|improve this answer










                          answered yesterday









                          Oldboy

                          7,1191832




                          7,1191832






















                              dfd is a new contributor. Be nice, and check out our Code of Conduct.










                              draft saved

                              draft discarded


















                              dfd is a new contributor. Be nice, and check out our Code of Conduct.













                              dfd is a new contributor. Be nice, and check out our Code of Conduct.












                              dfd is a new contributor. Be nice, and check out our Code of Conduct.
















                              Thanks for contributing an answer to Mathematics Stack Exchange!


                              • Please be sure to answer the question. Provide details and share your research!

                              But avoid



                              • Asking for help, clarification, or responding to other answers.

                              • Making statements based on opinion; back them up with references or personal experience.


                              Use MathJax to format equations. MathJax reference.


                              To learn more, see our tips on writing great answers.





                              Some of your past answers have not been well-received, and you're in danger of being blocked from answering.


                              Please pay close attention to the following guidance:


                              • Please be sure to answer the question. Provide details and share your research!

                              But avoid



                              • Asking for help, clarification, or responding to other answers.

                              • Making statements based on opinion; back them up with references or personal experience.


                              To learn more, see our tips on writing great answers.




                              draft saved


                              draft discarded














                              StackExchange.ready(
                              function () {
                              StackExchange.openid.initPostLogin('.new-post-login', 'https%3a%2f%2fmath.stackexchange.com%2fquestions%2f3060629%2fwhy-is-lim-m-to-infty-frac-sin-left-frac-pi2m-right-frac-pi2%23new-answer', 'question_page');
                              }
                              );

                              Post as a guest















                              Required, but never shown





















































                              Required, but never shown














                              Required, but never shown












                              Required, but never shown







                              Required, but never shown

































                              Required, but never shown














                              Required, but never shown












                              Required, but never shown







                              Required, but never shown







                              Popular posts from this blog

                              An IMO inspired problem

                              Management

                              Has there ever been an instance of an active nuclear power plant within or near a war zone?